Sequence & Series Level-2 Quiz-3

Sequence & Series Level-2 Quiz-3

Welcome to the Level – 2 Quiz – 3 of the topic Sequence & Series on Knowvation! The pattern of the quiz will be MCQ and you will encounter many challenging problems which are going to be very helpful for the preparation of various competitive exams like, CAT, IIFT, SNAP, XAT, TISSMAT, TISSNET, CMAT, SSC CGL, etc. You’re requested to please do read the instructions given below before starting this quiz.

Here are some basic instructions:

Time Limit: 10 minutes

Number of Multiple Choice Questions: 10

Passing marks: 70%

You’ll see the answers after you SUBMIT the quiz. ‘Green’ color ticks represents a correct answer and the ‘Red’ ticks represents a wrong answer

Please do provide your valuable feedback on the quiz or report any issue/mistake in the comment box below. ALL THE BEST! 😀

1.

If 20 is divided into four parts which are in A.P. such that the product of the first and fourth is to the product of the second and third in the ratio 2:3. Then find the largest number.

 
 
 
 

2. Three terms in arithmetic progression have a sum of 45 and a product of 3240. Find the smallest of them.

 
 
 
 

3. The sum to infinity of a G.P. is 27 and the sum of the squares of the terms is 243. Find the common ratio.

 
 
 
 

4. Given that |y| < 1, find the value of 3 + 6y + 9y^2 + 12y^3 + … ?

 
 
 
 

5. Find the sum of the series 1, 4/5, 16/25, 64/125 … infinite.

 
 
 
 

6. Sequence P is defined by pn = pn-1 + 3, p1 = 11, Sequence Q is defined as qn = qn-1 – 4, q3 = 103. If pk > qk+2, what is the smallest value k can take?

 
 
 
 

7. The value of sum of fi rst 75 terms of the sequence 150 × 2 + 148 × 4 + 146 × 6 + … is?

 
 
 
 

8. Find the sum to 15 terms of the series 3.4 2 + 4.5 2 + 5.6 2 + …

 
 
 
 

9. The number of common terms in the two sequences 17, 21, 25, …., 417 and 16, 21, 26, …., 466 is?

 
 
 
 

10. A group of 630 children is arranged in rows for a group photograph session. Each row contains three fewer children than the row in front of it. What number of rows is not possible?

 
 
 
 


Comments

No comments yet. Why don’t you start the discussion?

Leave a Reply

Your email address will not be published.